Представление функции интегралом Фурье

Интеграл Фурье как разложение в сумму гармоник

Интегральную формулу Фурье можно переписать следующим образом:
$$f\left(x\right)=\intop _{ 0 }^{ +\infty }{ \left[ a\left(\lambda \right)\cos { \lambda x } +b\left(\lambda \right)\sin { \lambda x } \right] d\lambda },\quad\left(\ast\right)$$ где
$$a\left(\lambda \right)=\frac { 1 }{ \pi } \intop _{ -\infty }^{ +\infty }{ f\left(\xi \right)\cos { \lambda \xi } d\xi } ,$$ $$b\left(\lambda \right)=\frac { 1 }{ \pi } \intop _{ -\infty }^{ +\infty }{ f\left(\xi\right)\sin { \lambda \xi } d\xi }.$$
Равенство $\left( \ast \right)$ аналогично разложению функции в тригонометрический ряд Фурье, а выражения $a\left(\lambda \right), b\left(\lambda \right)$ аналогичны формулам для коэффициентов Фурье.

Замечание. Для удобства дальнейших вычислений формула $\left(\ast\right)$ может быть упрощена, а именно:

  • Если $f\left(x\right)$ — чётная функция, то $$a\left(\lambda \right)=\frac { 2 }{ \pi } \intop_{ 0 }^{ +\infty }{ f\left(\xi\right)\cos { \lambda \xi } d\xi } ,$$ а $b\left(\lambda \right)$ принимает значение $0.$ Тогда формулу $\left(\ast\right)$ можно записать в виде $$f\left(x\right)=\frac { 2 }{ \pi } \intop_{ 0 }^{ +\infty }{ \cos { \lambda x } d\lambda } \intop_{ 0 }^{ +\infty }{ f\left(\xi\right)\cos { \lambda \xi } d\xi }.$$ Это выражение называется косинус-формулой Фурье.
  • Для нечётной $f\left(x\right)$ получаем соответственно, что $a\left(\lambda\right)$ обращается в нуль, а $$b\left(\lambda\right)=\frac { 2 }{ \pi } \intop_{ 0 }^{ +\infty }{ f\left(\xi\right)\sin { \lambda \xi } d\xi },$$ поэтому исходная формула приобретает вид $$f\left(x\right)=\frac { 2 }{ \pi } \intop_{ 0 }^{ +\infty }{ \sin { \lambda x } d\lambda } \intop_{ 0 }^{ +\infty }{ f\left(\xi\right)\sin { \lambda \xi } d\xi }.$$ Таким образом, мы получили синус-формулу Фурье.

Замечание. Интегральная формула Фурье имеет эквивалентную ей комплексную формулу интеграла Фурье $$f\left( x \right) =\frac { 1 }{ 2\pi } \int\limits _{ -\infty }^{ +\infty }{ d\lambda } \int _{ -\infty }^{ +\infty }{ f\left( \xi \right) { e }^{ i\lambda \left( x-\xi \right) }d\xi } .$$

Пример

Представить следующую функцию интегралом Фурье: $$f\left(x\right)=\begin{cases} 1,\quad если \quad \left| x \right| < 1; \\ 0,\quad если \quad \left| x \right| > 1. \end{cases}$$

Решение

Данная функция удовлетворяет достаточным условиям, а потому её можно представить в виде интеграла Фурье. Построим график данной функции. fourier_integral_example_1 Он симметричен относительно оси ординат, следовательно, исходная функция — чётная. Опираясь на замечание для первого случая, имеем: $$b\left(\lambda\right)=0,$$ $$a\left(\lambda \right)=\frac { 2 }{ \pi } \intop_{ 0 }^{ +\infty }{ f\left(t\right)\cos { \lambda t } dt }=\frac { 2 }{ \pi } \intop_{ 0 }^{ 1 }{ f\left(t\right)\cos { \lambda t } dt }=\frac { 2\sin { \lambda } }{ \pi \lambda }.$$ Подставив результат вычислений $a\left(\lambda \right)$ в формулу интеграла Фурье получаем ответ: $$f\left(x\right)=\frac { 2 }{ \pi } \intop_{ 0 }^{ +\infty }{ \frac { \sin { \lambda } }{ \lambda } } \cos { \lambda x } d\lambda, \quad \left| x \right|\neq 1.$$

[свернуть]

Литература

Тестирование. Представление функции интегралом Фурье

Тесты помогут понять насколько хорошо был усвоен материал.

Определение интеграла Фурье

Для лучшего понимания материала, изложенного ниже, пожалуйста, ознакомьтесь с темой «Ряды Фурье».

Интегральная формула Фурье

Если интервал $\left[ -l,l \right],$ на котором функция $f\left(x\right)$ разлагается в тригонометрический ряд Фурье, неограниченно возрастает, т.е. $l\rightarrow +\infty,$ то ряд Фурье превращается в интеграл Фурье. При переходе к пределу происходит качественный скачок: функция, заданная на любом конечном интервале $\left[ -l,l \right],$ разлагается в ряд «гармонических колебаний», частоты которых образуют дискретную последовательность; функция $f\left(x\right),$ заданная на всей оси $x$ или на полуоси $x,$ разлагается в интеграл, который представляет собой сумму «гармонических колебаний», частоты которых непрерывно заполняют действительную полуось $0\le \lambda \le +\infty .$ Рассмотрим этот предельный переход от ряда Фурье к интегралу Фурье.

Замечание. Напомним, что функция $f$ является кусочно-гладкой на отрезке $\left[ a,b \right],$ если:

  • $f$ непрерывна во всех точках, кроме, быть может, конечного числа точек ${ x }_{ 1 },\dots ,{ x }_{ n }\in \left(a,b\right).$
  • $\forall i=1,\dots ,n \quad \exists f\left({ x }_{ i }\pm 0\right),\quad f\left(a+0\right),\quad f\left(b-0\right).$
  • $f$ – дифференцируема во всех точках, кроме, быть может, конечного числа точек ${ x }_{ 1 },\dots ,{ x }_{ n }.$
  • $\exists f^{ \prime }\left({ x }_{ i }\pm 0\right).$Пусть $f\left(x\right)$ задана на всей оси $x$ и на каждом конечном отрезке $\left[ -l,l \right],$ является кусочно-гладкой. Тогда, в силу основной теоремы о сходимости тригонометрического ряда Фурье, при любом $l>0$ $$f(x)=\frac { { a }_{ 0 } }{ 2 } +\sum _{ k=1 }^{ +\infty }{ \left( { a }_{ k }\cos { \frac { k\pi x }{ l } } +{ b }_{ k }\sin { \frac { k\pi x }{ l } } \right) } ,\quad \left( 1 \right) $$
    где $$\left(2\right)\quad \begin{cases} { a }_{ 0 }=\frac { 1 }{ l } \int\limits_{ -l }^{ l }{ f\left(\xi \right) } d\xi , \\ { a }_{ k }=\frac { 1 }{ l } \int\limits_{ -l }^{ l }{ f\left(\xi \right)\cos { \frac { k\pi \xi }{ l } d\xi , } } \\ { b }_{ k }=\frac { 1 }{ l } \int\limits_{ -l }^{ l }{ f\left(\xi \right)\sin { \frac { k\pi \xi }{ l } d\xi . } } \end{cases}$$
    Равенство $\left(1\right)$ имеет место, если $x$ — внутренняя точка отрезка $\left[ -l,l \right],$ в которой $f\left(x\right)$ непрерывна; если же $x$ — внутренняя точка этого отрезка, в которой $f\left(x\right)$ разрывна, то в левой части равенства $\left(1\right)$ $f\left(x\right)$ нужно заменить через $\frac { f\left(x+0\right)+f\left(x-0\right) }{ 2 }.$
    Подставляя выражения $\left(2\right)$ в $\left(1\right),$ получим $$f\left(x\right)=\frac { 1 }{ 2l } \intop_{ -l }^{ l }{ f\left(\xi \right)d\xi } +\frac { 1 }{ l } \sum _{ k=1 }^{ +\infty }{ \intop_{ -l }^{ l }{ f\left(\xi \right)\cos { \frac { k\pi }{ l } } \left(\xi -x\right)d\xi } }.\quad \left(3\right) $$
    Если $f\left(x\right)$ ещё и абсолютно интегрируема на всей оси $x,$ т.е. $$\intop_{ -\infty }^{ +\infty }{ \left| f\left(x\right) \right| dx } =Q<+\infty, \quad \left(4\right)$$
    то при переходе к пределу при $l\rightarrow +\infty$ первое слагаемое в правой части $\left(3\right)$ в силу условия $\left(4\right)$ стремится к нулю. Следовательно, $$f\left(x\right)=\lim _{ l\rightarrow +\infty }{ \frac { 1 }{ l } \sum _{ k=1 }^{ +\infty }{ \intop_{ -l }^{ l }{ f\left(\xi \right)\cos { \frac { k\pi }{ l } } \left(\xi -x\right)d\xi } . } } \quad \left(5\right)$$ Положим $\frac { k\pi }{ l } ={ \lambda }_{ k },$ $\frac { \pi }{ l } ={ \Delta \lambda }_{ k }.$ Тогда $\left(5\right)$ можно переписать в виде $$f\left( x \right) =\lim _{ \begin{matrix} l\rightarrow +\infty \\ \Delta { \lambda }_{ k }\rightarrow 0 \end{matrix} }{ \frac { 1 }{ \pi } } \sum _{ k=1 }^{ +\infty }{ \Delta { \lambda }_{ k } } \intop_{ -l }^{ l }{ f\left( \xi \right) \cos { { \lambda }_{ k } } \left( \xi -x \right) d\xi }.\quad \left( 6 \right) $$
    Будем рассуждать нестрого:

    1. при больших значениях $l$ интеграл $$\intop_{ -l }^{ l }{ f\left(\xi \right)\cos { { \lambda }_{ k } } \left(\xi -x\right)d\xi }$$ можно заменить интегралом
      $$\intop_{ -\infty }^{ +\infty }{ f\left(\xi \right)\cos { { \lambda }_{ k } } \left(\xi -x\right)d\xi },$$
    2. $$\sum _{ k=1 }^{ +\infty }{ \Delta { \lambda }_{ k } } \intop_{ -\infty }^{ +\infty }{ f\left(\xi \right)\cos { { \lambda }_{ k } } \left(\xi -x\right)d\xi } $$ является интегральной суммой для интеграла $$\intop_{ 0 }^{ +\infty }{ d\lambda } \intop_{ -\infty }^{ +\infty }{ f\left(\xi \right)\cos { { \lambda } } \left(\xi -x\right)d\xi } ,$$ поэтому из $\left(6\right)$ получаем $$f\left(x\right)=\frac { 1 }{ \pi } \intop_{ 0 }^{ +\infty }{ d\lambda } \intop_{ -\infty }^{ +\infty }{ f\left(\xi \right)\cos { { \lambda } } \left(\xi -x\right)d\xi } , \quad \left(7\right)$$ где в левой части равенства $\left(7\right)$ вместо $f\left(x\right)$ нужно писать $\frac { f\left(x+0\right)+f\left(x-0\right) }{ 2 } ,$ если $x$ является точкой разрыва функции $f\left(x\right).$

    Равенство $\left(7\right)$ называется интегральной формулой Фурье, а интеграл, стоящий в её правой части, — интегралом Фурье либо двойным интегралом Фурье

    Обоснование интегральной формулы Фурье

    Равенство $\left(7\right)$ было получено с помощью формальных предельных переходов, которые не были обоснованы.
    Вместо того чтобы их обосновать, удобнее непосредственно доказывать справедливость равенства $\left(7\right).$

    Теорема

    Если функция $f\left(x\right),$ кусочно-гладкая на каждом конечном отрезке оси $x,$ абсолютно интегрируема на всей оси $x,$ т.е. интеграл $\int\limits_{ -\infty }^{ +\infty }{ \left| f\left(x\right) \right| dx } $ сходится, то $$\lim _{ l\rightarrow +\infty }{ \frac { 1 }{ \pi } \intop_{ 0 }^{ l }{ d\lambda } \intop_{ -\infty }^{ +\infty }{ f\left(\xi \right)\cos { { \lambda } } \left(\xi -x\right)d\xi } } =\frac { f\left(x+0\right)+f\left(x-0\right) }{ 2 }.$$

    Доказательство

    Заметим прежде всего, что интеграл $$\intop_{ -\infty }^{ +\infty }{ f\left(\xi \right)\cos { \lambda \left(\xi -x\right)d\xi } },$$ зависящий от параметра $\lambda,$ сходится равномерно по параметру $\lambda$ при $0\le \lambda \le +\infty,$ так как $\left| f\left(\xi \right)\cos { \lambda } \left(\xi -x\right) \right| \le \left| f\left(\xi \right) \right| ,$ а интеграл $\int\limits_{ -\infty }^{ +\infty }{ \left| f\left(\xi \right) \right| d\xi } $ по условию сходится. Следовательно, можно изменить порядок интегрирования, т.е. записать так:
    $$\frac { 1 }{ \pi } \intop_{ 0 }^{ l }{ d\lambda } \intop_{ -\infty }^{ +\infty }{ f\left(\xi \right)\cos { { \lambda } } \left(\xi -x\right)d\xi } =$$
    $$=\frac { 1 }{ \pi } \intop_{ -\infty }^{ +\infty }{ d\xi } \intop_{ 0 }^{ l }{ f\left(\xi \right)\cos { { \lambda } } \left(\xi -x\right)d\lambda } =$$
    $$=\frac { 1 }{ \pi } \intop_{ -\infty }^{ +\infty }{ f\left(\xi \right)\frac { \sin { l\left(\xi -x\right) } }{ \xi -x } d\xi } =$$
    $$=\frac { 1 }{ \pi } \intop_{ -\infty }^{ +\infty }{ f\left(x+\zeta \right)\frac { \sin { l\zeta } }{ \zeta } d\zeta } ,$$
    где $\zeta=\xi-x,$ $d\zeta=d\xi.$ Нам остаётся доказать, что $$\lim _{ l\rightarrow +\infty }{ \frac { 1 }{ \pi } \intop_{ -\infty }^{ 0 }{ f\left(x+\zeta \right)\frac { \sin { l\zeta } }{ \zeta } d\zeta } } =\frac { f\left(x-0\right) }{ 2 },\quad\left(8\right)$$
    $$\lim _{ l\rightarrow +\infty }{ \frac { 1 }{ \pi } \intop_{ 0 }^{ +\infty }{ f\left(x+\zeta \right)\frac { \sin { l\zeta } }{ \zeta } d\zeta } } =\frac { f\left(x+0\right) }{ 2 }.\quad\left(9\right)$$
    При доказательстве мы воспользуемся известным соотношением (см. п. 5 § 2 гл. 10) $$\frac { 1 }{ \pi } \intop_{ 0 }^{ +\infty }{ \frac { \sin { l\zeta } }{ \zeta } d\zeta } =\frac { 1 }{ 2 } \quad \left(10\right).$$ Докажем, например, справедливость соотношения $\left(9\right).$ В силу равенства $\left(10\right),$ можно записать, что $$\frac { f\left(x+0\right) }{ 2 } =\frac { 1 }{ \pi } \intop_{ 0 }^{ +\infty }{ f\left(x+0\right)\frac { \sin { l\zeta } }{ \zeta } d\zeta } .$$
    Поэтому разность между переменной величиной и предполагаемым пределом в соотношении $\left(9\right)$ будет равна
    $${ J }_{ 0,+\infty }=\frac { 1 }{ \pi } \intop_{ 0 }^{ +\infty }{ f\left(x+ \zeta \right)\frac { \sin { l\zeta } }{ \zeta } d\zeta } -\frac { f\left(x+0\right) }{ 2 } =$$
    $$=\frac { 1 }{ \pi } \intop_{ 0 }^{ +\infty }{ \left[ f\left(x+\zeta \right)-f\left(x+0\right) \right] \frac { \sin { l\zeta } }{ \zeta } d\zeta } .\quad\left(11\right)$$
    Таким образом, нужно доказать, что этот интеграл стремится к нулю при $l\rightarrow +\infty.$ Разобьём интервал интегрирования $0\le \zeta <+\infty $ на три:
    $0 < \zeta \le\delta ,$ $\quad \delta \le \zeta \le\Delta ,$ $\quad \Delta \le \zeta <+\infty ;$ тогда интеграл $\left(11\right)$ будет представлен в виде суммы трёх интегралов $$ { J }_{ 0,+\infty }={ J }_{ 0,\delta }+{ J }_{ \delta ,\Delta }+{ J }_{ \Delta ,+\infty }. \quad\left(12\right)$$ После этого будем действовать следующим образом. Сначала, задавшись произвольным $\varepsilon >0,$ докажем, что при всех достаточно малых $\delta>0$ и всех достаточно больших $\Delta >\delta$ будут выполняться неравенства $$\left| { J }_{ 0,\delta } \right| <\frac { \varepsilon }{ 3 }\quad и \quad \left| { J }_{ \Delta,+\infty } \right| <\frac { \varepsilon }{ 3 } \quad \left(13\right)$$ сразу при всех $l\ge 1.$ Затем, фиксировав $\delta$ и $\Delta$ так, чтобы выполнялись неравенства $\left(13\right),$ выберем $l\ge 1$ столь большим, чтобы в силу основной леммы выполнялось неравенство $\left| { J }_{ \delta ,\Delta } \right| <\frac { \varepsilon }{ 3 } .$ Отсюда, в силу $\left(12\right),$ будет следовать, что $\left| { J }_{ 0,+\infty } \right| <\varepsilon $ при всех достаточно больших $l\ge 1.$ Итак, оценим сначала интеграл $${ J }_{ 0,\delta }=\frac { 1 }{ \pi } \intop_{ 0 }^{ \delta }{ \frac { f\left(x+\zeta \right)-f\left(x+0\right) }{ \zeta } \sin { l\zeta } d\zeta } .$$ При всех достаточно малых $\delta>0$ $$\left| \frac { f\left(x+\zeta \right)-f\left(x+0\right) }{ \zeta } \right| <\left| { f }_{ + }^{ \prime }\left(x\right) \right| +1\quad \forall \zeta \in \left(0,\delta \right).$$ Следовательно, $$\left| { J }_{ 0,\delta } \right| <\frac { \delta }{ \pi } \left\{ \left| { f }_{ + }^{ \prime }\left(x\right) \right| +1 \right\} <\frac { \varepsilon }{ 3 } \quad\left(14\right)$$ при всех $\delta <\frac { \varepsilon \pi }{ 3\left\{ \left| { f }_{ + }^{ \prime }\left( x \right) \right| +1 \right\} } $ и при всех значениях $l.$ Оценим, далее, интеграл $${ J }_{ \Delta ,+\infty }=\frac { 1 }{ \pi } \intop_{ \Delta }^{ +\infty }{ f\left(x+\zeta \right)\frac { \sin { l\zeta } }{ \zeta } d\zeta } -\frac { f\left(x+0\right) }{ \pi } \intop_{ \Delta }^{ +\infty }{ \frac { \sin { l\zeta } }{ \zeta } d\zeta } .$$ Мы имеем $$\left| { J }_{ \Delta ,+\infty } \right| \le \frac { 1 }{ \pi } \intop_{ \Delta }^{ +\infty }{ \left| f\left(x+\zeta \right) \right| \frac { d\zeta }{ \zeta } } +\frac { \left| f\left(x+0\right) \right| }{ \pi } \left| \intop_{ \Delta }^{ +\infty }{ \frac { \sin { l\zeta } }{ \zeta } d\zeta } \right| \le $$ $$\le \frac { 1 }{ \pi \Delta } \intop_{ -\infty }^{ +\infty }{ \left| f\left(x+\zeta \right) \right| d\zeta } +\frac { \left| f\left(x+0\right) \right| }{ \pi } \left| \intop_{ l\Delta }^{ +\infty }{ \frac { \sin { { \zeta }^{ \ast } } }{ { \zeta }^{ \ast } } } d{ \zeta }^{ \ast } \right| =$$
    $$=\frac { Q }{ \pi \Delta } +\frac { \left| f\left(x+0\right) \right| }{ \pi } \left| \intop_{ l\Delta }^{ +\infty }{ \frac { \sin { { \zeta }^{ \ast } } }{ { \zeta }^{ \ast } } d{ \zeta }^{ \ast } } \right| ,$$ где ${ \zeta }^{ \ast }=l\zeta. \quad\left(15\right)$ Напомним, что, согласно условию $\left(4\right),$ $Q=\int\limits_{ -\infty }^{ +\infty }{ \left| f\left(x\right) \right| dx } <\infty,$ поэтому при всех достаточно больших $\Delta>0$ будет $\frac { Q }{ \pi \Delta } <\frac { \varepsilon }{ 6 } $ сразу для всех $l.$ Далее, так как интеграл $\int\limits_{ 0 }^{ +\infty }{ \frac { \sin { { \zeta }^{ \ast } } }{ { \zeta }^{ \ast } } d{ \zeta }^{ \ast } } $ сходится, то при всех достаточно больших $\Delta>0$ и всех $l\ge 1$ $$\frac { \left| f\left(x+0\right) \right| }{ \pi } \left| \intop_{ l\Delta }^{ +\infty }{ \frac { \sin { { \zeta }^{ \ast } } }{ { \zeta }^{ \ast } } d{ \zeta }^{ \ast } } \right| <\frac { \varepsilon }{ 6 } .$$ Следовательно, в силу $\left(15\right)$ $$\left| { J }_{ \Delta ,+\infty } \right| <\frac { \varepsilon }{ 3 } \quad\left(16\right)$$ при всех достаточно больших $\Delta>0$ и всех $l\ge 1.$ Оценим, наконец, интеграл $${ J }_{ \delta ,\Delta }=\frac { 1 }{ \pi } \intop_{ \delta }^{ \Delta }{ \frac { f\left(x+\zeta \right)-f\left(x+0\right) }{ \zeta } \sin { l\zeta } d\zeta } .$$ Функция $\frac { f\left(x+\zeta \right)-f\left(x+0\right) }{ \zeta } $ по переменной $\zeta$ является кусочно-гладкой на отрезке $\delta \le \zeta \le \Delta .$ Поэтому, в силу основной леммы, при всех достаточно больших значениях $l\ge1$ будет выполняться неравенство $$\left| { J }_{ \delta ,\Delta } \right| <\frac { \varepsilon }{ 3 }. \quad\left(17\right)$$ Сопостовляя $\left(14\right), \left(16\right)$ и $\left(17\right),$ получим, что при всех достаточно больших $l\ge1$ $$\left| { J }_{ 0,+\infty } \right| <\varepsilon ,$$ что и требовалось доказать. $\blacksquare$

    [свернуть]

    Замечание. Основная теорема об интеграле Фурье справедлива и при более слабых ограничениях, налагаемых на функцию $f\left(x\right).$ А именно, если абсолютно интегрируемая на всей оси $x$ функция $f\left(x\right)$

    • кусочно-непрерывна на каждом конечном отрезке оси $x$
    • отношение $\left| \frac { f\left(x+\zeta \right)-f\left(x+0\right) }{ \zeta } \right|$ ограничено при любом фиксированном $x$ для всех достаточно малых $\zeta,$ то основная теорема сохраняет силу.
    Доказательство

    Действительно, доказательство основной теоремы сводится к оценке трёх интегралов: ${ J }_{ 0,\delta },{ J }_{ \delta ,\Delta },{ J }_{ \Delta ,+\infty }$ для ${ J }_{ 0 ,+\infty }.$ Последний из этих трёх интегралов мал при достаточно большом $\Delta,$ в силу абсолютной интегрируемости $f\left(x\right).$ Интеграл ${ J }_{ 0,\delta }$ мал при всех достаточно малых $\delta>0,$ если отношение $\left| \frac { f\left(x+\zeta \right)-f\left(x+0\right) }{ \zeta } \right| $ ограничено при каждом фиксированном $x$ для всех достаточно малых $\zeta>0.$ В интеграле же $${ J }_{ \delta ,\Delta }=\frac { 1 }{ \pi } \intop _{ \delta }^{ \Delta }{ \frac { f\left(x+\zeta \right)-f\left(x+0\right) }{ \zeta } \sin { l\zeta } d\zeta } $$ функция $\varphi \left(\zeta \right)= \frac { f\left(x+\zeta \right)-f\left(x+0\right) }{ \zeta } $ кусочно-непрерывна на отрезке $0<\delta \le \zeta \le \Delta $ при любом фиксированном $x.$ Пусть $\left[ a,b \right] $ — какой-либо сегмент, на котором $\varphi \left(\zeta \right)$ непрерывна, и пусть дано какое угодно $\varepsilon>0.$ Построим такую кусочно-гладкую функцию ${ g }_{ \varepsilon }\left(x\right)$ (как при доказательстве первой теоремы Вейерштрасса), чтобы выполнялось неравенство $$\left| \varphi \left(\zeta \right)-{ g }_{ \varepsilon }\left(\zeta\right) \right| <\frac { \varepsilon }{ 2\left(b-a\right) },\quad 0<\delta \le \zeta \le \Delta .$$ Но тогда $$\left| \int _{ a }^{ b }{ \varphi \left(\zeta \right)\sin { l\zeta } d\zeta } \right| \le \intop _{ a }^{ b }{ \left| \varphi \left(\zeta \right)-{ g }_{ \varepsilon }\left(\zeta\right) \right| d\zeta } +$$ $$+\left| \intop _{ a }^{ b }{ { g }_{ \varepsilon }\left(\zeta \right)\sin { l\zeta } d\zeta } \right| <\frac { \varepsilon }{ 2 } +\frac { \varepsilon }{ 2 } =\varepsilon \quad $$ при всех достаточно больших $l\ge0,$ так как для кусочно-гладкой функции ${ g }_{ \varepsilon }\left(x\right)$ справедлива основная лемма. Разбивая интеграл $ { J }_{ \delta ,\Delta }$ на интервалы по сегментам непрерывности $\varphi \left(\zeta \right),$ получаем, что ${ J }_{ \delta ,\Delta }\rightarrow 0$ при $l\rightarrow +\infty,$ чем и завершается доказательство теоремы.

    [свернуть]

    Литература

    Тестирование. Интеграл Фурье

    После прочтения материала настоятельно рекомендую попробовать силы в несложных тестах для закрепления материала.
    Желаю успехов!

Несобственные интегралы от неограниченных функций

Определение

Пусть функция [latex]f[/latex] задана на полуинтервале [latex][a,b)[/latex], где $-\infty<a<b<+\infty$, и интегрируема по Риману на любом отрезке [latex][a,\xi][/latex], где $a<\xi<b$. Тогда, если существует конечный предел [latex]\lim_{\xi \to b-0}\int_{a}^{\xi}{f(x)dx}[/latex], то несобственный интеграл $II$ рода [latex]\int_{a}^{b}{f(x)dx}[/latex] называют сходящимся и полагают

$$\int\limits_a^b{f(x)dx}=\lim_{\xi \to b-0}\int\limits_{a}^{\xi}{f(x)dx}$$

В противном случае несобственный интеграл называют расходящимся.

Аналогично, если существует конечный [latex]\lim_{\xi \to a+0}\int_{\xi}^{b}{f(x)dx}[/latex], то несобственный интеграл $II$ рода [latex]\int_{a}^{b}{f(x)dx}[/latex] называют сходящимся и полагают

$$\int\limits_a^b{f(x)dx}=\lim_{\xi \to a+0}\int\limits_{\xi}^{b}{f(x)dx}$$

В противном случае, если такого предела нет, расходящимся.

Замечание

Определение несобственного интеграла от непрерывных функций является содержательным лишь в случае, когда  [latex]f(x)[/latex] неограниченна  в окрестности точек [latex]b,a[/latex]. При этом, эти точки называются особыми.

Пример:

Курсовая
Рассмотрим функцию [latex]\frac{1}{\sqrt{1-x}}[/latex]. Эта функция непрерывна на промежутке [latex][0,1)[/latex], но не ограничена на этом промежутке. При [latex]\forall\xi\in [0,1)[/latex] функция [latex]\frac{1}{\sqrt{1-x}}[/latex] интегрируема на отрезке [latex][0,\xi][/latex], причем [latex]J(\xi)=\int_{0}^{\xi}{\frac{dx}{\sqrt{1-x}}}=\left(-2\sqrt{1-x})\right|^{\xi}_{0}=2(1-\sqrt{1-\xi})[/latex], откуда следует, что существует конечный [latex]\lim_{\xi \to 1-0}F(\xi)=2[/latex]. В этом случае говорят, что несобственный интеграл от функции [latex]\frac{1}{\sqrt{1-x}}[/latex] на промежутке [latex][0,1)[/latex] равен [latex]2[/latex], т.е. [latex]\int_{0}^{1}{\frac{dx}{\sqrt{1-x}}}=2[/latex]. Число [latex]2[/latex] можно интерпретировать как площадь заштрихованной фигуры на Рис.1.

Тест по теме: Несобственные интегралы от неограниченных функций

Этот тест покажет насколько хорошо вы усвоили данную тему.

Таблица лучших: Тест по теме: Несобственные интегралы от неограниченных функций

максимум из 10 баллов
Место Имя Записано Баллы Результат
Таблица загружается
Нет данных

Оценка погрешности приближенного вычисления определенных интегралов по формуле Тейлора

Рассмотрим погрешность приближённого вычисления определённых интегралов по формуле Тейлора.

Обозначим погрешность через [latex]R_{n}[/latex]

[latex]R_{n}[/latex] представляет собой разность истинного значения определённого интеграла и полученного в результате приблизительного вычисления.

Разумеется, что истинное значение также считается приближённо. Иначе, можно было б использовать точные методы вычисления определённых интегралов.

Проанализируем погрешность вычисление примера 1 :

[latex]\int_{0}^{0.3} e^{-2x^{2}}=0.3-2\frac{0.3^{3}}{3}+2\frac{0.5^{5}}{5}-\frac{4*(0.3)^{7}}{21}+…=0.3-0.018+0.000972-…\approx[/latex]

[latex] \approx0.3-0.018=0.282[/latex]

Видем, что каждый следующий член суммы на порядки меньше предыдущего.

Если вычислить интеграл, взяв только первый член ряда, получим погрешность [latex]R_{n}\approx0.018972[/latex]

Два первых:

[latex]R_{n}\approx0.000972[/latex]

Имеем, что высокая точность достигается довольно быстро.

Аналогичные рассуждения можно провести с  примером 2.

Литература :

Интегрирование рациональных функций от sin x, cos x и sinh x, cosh x. Универсальная подстановка.

Интегрирование любого рационального выражения тригонометрических функций можно всегда свести к интегрированию алгебраической рациональной функции используя универсальную тригонометрическую подстановку $latex x=2\arctan t$    или  $latex \tan \frac{x}{2}=t$ .

Интегралы вида $latex \int R(\sin x, \cos x)dx$   , где R-рациональная функция.

В результате подстановки   $latex t=\tan \frac{x}{2}$    в указанные интегралы получаем:

$latex \sin x=\frac{2\tan \frac{x}{2}}{1+\tan ^{2}\frac{x}{2}}=\frac{2t}{1+t^{2}}$ ;       $latex \cos x=\frac{1-\tan ^{2}\frac{x}{2}}{1+\tan ^{2}\frac{x}{2}}=\frac{1-t^{2}}{1+t^{2}}$ , где    $latex dx=\frac{2dt}{1+t^{2}}$ .

Гиперболические функции    определяются следующим образом:

$latex \sinh x=\frac{e^{x}-e^{-x}}{2}$ ;       $latex \cosh x=\frac{e^{x}+e^{-x}}{2}$ .


Приведем еще несколько полезных соотношений :   

  • $latex \cosh ^{2}x-\sinh ^{2}x=1$ ;
  • $latex \sinh 2x=2\sinh \cosh $ ;
  • $latex \cosh 2x=\cosh ^{2}+\sinh ^{2} $

Если подынтегральное выражение содержит гиперболическую функцию, то такой интеграл можно свести к интегрированию рациональной функции с помощью подстановки 

$latex t=e^{x}$ ;           $latex x=\ln t$ ;           $latex dx=\frac{dt} {t}$ .

Рассмотрим несколько примеров:

(Прочитав вышеизложенный материал, попробуйте решить следующие примеры. Если же решить не получиться, жмите «ПОКАЗАТЬ»)

 

1) Найти интеграл $latex \int \frac{dx}{4\sin x+3\cos x+5}$

Подсказка: используйте подстановку        $latex \tan \frac{x}{2}=t$

Спойлер

\small \inline \dpi{100} \fn_jvn \Delta Подынтегральная функция рационально зависит от  $latex \sin x$  и  $latex \cos x$; применим подстановку $latex \tan \frac{x}{2}=t$,

тогда  $latex \sin x=\frac{2t}{1+t^{2}}$ ;  $latex \cos x=\frac{1-t^{2}}{1+t^{2}}$ ;  $latex dx=\frac{2dt}{1+t^{2}}$       и

$latex \int \frac{dx}{4\sin x+3\cos x+5}=$ $latex \int \frac{\frac{2dt}{1+t^{2}}}{4\cdot \frac{2t}{1+t^{2}}+3\cdot \frac{1-t^{2}}{1+t^{2}}}=$

$latex =2 \int \frac{dt}{2t^{2}+8t+5}=$  $latex \int \frac{dt}{(t+2)^{2}}=$ $latex =-\frac{1}{t+2}+C$ .

Возвращаясь к старой переменной, получим

$latex \int \frac{dx}{4\sin x+3\cos x+5}=-\frac{1}{\tan \frac{x}{2}}+C$   $latex \blacktriangle$

[свернуть]

 

 

2) Найти интеграл $latex \int \frac{(\sin x+\sin ^{3}x)dx}{\cos 2x}$ .

Подсказка : используйте замену   $latex \cos x=t$   , а также свои знания по теме  «Тригонометрические тождества» 

Спойлер

$latex \triangle$ Так как подынтегральная функция нечетна относительно синуса, то полагаем $latex \cos x=t$.

Отсюда    $latex \sin ^{2}x=1-t^{2},\ \cos 2x=2\cos ^{2}x-1=2t^{2}-1,\ dt=-\sin x \ dx.$

Таким образом :

$latex \int \frac{(\sin x+\sin ^{3}x)dx}{\cos 2x}=\int \frac{(2-t^{2})(-dt)}{2t^{2}-1}=\int \frac{(2t^{2}-2)\ dt}{2t^{2}-1}=$

$latex =\frac{1}{2}\int \frac{2t^{2}-4}{2t^{2}-1}\ dt=\frac{1}{2}\int dt-\frac{3}{2}\int\frac{dt}{2t^{2}-1}=$

$latex =\frac{t}{2}-\frac{3}{2\sqrt{2}}\int \frac{d(t\sqrt{2})}{2t^{2}-1}=\frac{t}{2}-\frac{3}{2\sqrt{2}}\ln \left | \frac{t\sqrt{2}-1}{t\sqrt{2}+1} \right |+C.$

Следовательно:

$latex \int \frac{(\sin x+\sin ^{3}x)dx}{\cos 2x}=\frac{1}{2}\cos x-\frac{3}{2\sqrt{2}} \ln\left | \frac{\sqrt{2}\cos x-1}{\sqrt{2}\cos x+1} \right |+C .$          $latex \blacktriangle$

[свернуть]

 

 

3) Найти интеграл $latex \int \frac{\cosh x}{2+3\sinh x}dx$

Подсказка: используйте подстановку    $latex t=2+3\sinh x $ 

Спойлер

$latex \triangle$ Сделаем подстановку $latex t=2+3\sinh x,\ du=3\cosh xdx.$ Тогда $latex \cosh xdx=\frac{dt}{3}.$ Следовательно, интеграл равен

$latex \int \frac{\cosh x}{2+3\sinh x}dx=\int \frac{dt}{3}\cdot \frac{1}{t}=\frac{1}{3}\int \frac{dt}{t}=\frac{1}{3}\ln \left | t \right |+C=\frac{1}{3}\ln \left | 2+3\sinh x \right |+C.$       $latex \blacktriangle$

[свернуть]

 

 

4) Найти интеграл $latex \int \sinh ^{3}xdx$
Подсказка:  используйте гиперболиские соотношения 

Спойлер

$latex \triangle$ Поскольку $latex \cosh ^{2}x-\sinh ^{2}x=1$, и, следовательно, $latex \sinh ^{2}x=\cosh ^{2}x-1,$ интеграл можно переписать в виде

$latex \mathbb{I}=\int \sinh ^{3} xdx=\int \sinh ^{2}x\cosh xdx=\int (\cosh ^{2}x-1)\sinh xdx$

Делая замену $latex t=\cosh x,\ dt=\sinh xdx,$ получаем

$latex \mathbb{I}=\int (\cosh ^{2}x-1)\sinh xdx=\int (t^{2}-1)dt=$

$latex =\frac{t^{3}}{3}-t+C=\frac{\cosh ^{3}x}{3}-\sinh x+C$ $latex \blacktriangle$

[свернуть]

Литература:

  • А.Г. Попов, П.Е. Данко, Т.Я. Кожевникова (издание 6-е часть 1) стр. 234-242
  • Конспекты по мат.анализу (преп. Лысенко З.М.)
  • Ещё больше примеров можно найти  здесь

Дополнительные материалы :

  • Лекции по матанализу т1. стр. 171-173
  • Г.М.Фихтенгольц т.2  1964 год стр. 73-78

 

 

 

Тест (Вычисление интегралов методом универсальной подстановки)

по темам «Интегрирование рациональных функций от sin x, cos x и sinh x, cosh x» и «Универсальная подстановка«


Таблица лучших: Тест (Вычисление интегралов методом универсальной подстановки)

максимум из 7 баллов
Место Имя Записано Баллы Результат
Таблица загружается
Нет данных